Q10

 
joliebella
Thanks Received: 1
Vinny Gambini
Vinny Gambini
 
Posts: 10
Joined: June 17th, 2010
 
 
 

Q10

by joliebella Wed Jun 01, 2011 3:35 pm

This was the only question that stumped me. It was the last one I answered; and I guess I just did not put much effort into understanding the ques.

At first, I picked A, then B both are wrong. The correct answer is C, but I don't understand why O needs to be determined in order to identify who lectures that week. Can you provide a detailed explanation and perhaps a diagram? I would greatly appreciate this.
User avatar
 
ManhattanPrepLSAT1
Thanks Received: 1909
Atticus Finch
Atticus Finch
 
Posts: 2851
Joined: October 07th, 2009
 
 
 

Re: Q10

by ManhattanPrepLSAT1 Fri Jun 03, 2011 2:43 pm

What we have at the start is K and L divided between weeks 1 and 2. M and N are also divided between weeks 3 and 4 at the start. Finally we are not certain whether O or P is in week 5.

To completely determine the schedule we would need to know where one of K and L goes, so that we could determine the other, where one of M and N goes, so that we could determine the other, and finally whether it is O or P in week 5.

Answer choice (C) is perfect because it gives us one of K and L, one of M and N, and one of O and P.

(A) would leave week 5 undetermined.
(B) would leave week 5 undetermined.
(D) would leave weeks 1 and 2 undetermined.
(E) would leave weeks 1 and 2 undetermined.

I hope that helps!